2021年度高考数学問題北京版

以下,2021年度の高考の問題を紹介します.
公表されている問題および解答は次をご覧ください.

中国の大学入試問題2021(北京版)

解答もついています.当然中国語ですが,問題を見てから読めば十分理解できます.

2021年 全国統一試験,数学(理系)北京版

この問題は5ページよりなる.
配点150点.制限時間 120分.
受験生は必ず答えを解答欄に記入すること.答案用紙上のそれ以外の解答は無効である.
試験終了後,問題用紙,解答用紙はともに回収する.

選択問題で12問よりなる.配点は各問5点合計60点.
各設問のなかの4つの選択肢より求められた符号を一つ選びなさい.


$1.$ 集合$M=\{x|-4 < x < 2\},N=\{x|x^2-x-6 < 0\}$のとき$M\cap N=$
$(A) \ \{x|-4 < x < 3\}$
$(B) \ \{x|-4 < x < -2\}$
$(C) \ \{x|-2 < x < 2\}$
$(D) \ \{x|2 < x < 3\}$

$2.$複素数$z$が$|z-i|=1$を満たすとき,$z$の複素平面内における対応点を$(x,y)$とすると,
$(A) \ (x+1)^2+y^2=1$
$(B) \ (x-1)^2+y^2=1$
$(C) \ x^2+(y-1)^2=1$
$(D) \ x^2+(y+1)^2=1$

$3. \ a=\log _20.2 \ ,b=2^{0.2} \ ,c=0.2^{0.3}$のとき
$(A) \ a < b < c$
$(B) \ a < c < b$
$(C) \ c < a < b$
$(D) \ b < c < a$

$4. \ $古代ギリシャ時代,最も美しい人体は頭の頂上からへそまでの長さとへそから足底までの長さの比が$\frac{\sqrt{5}-1}{2}$($\frac{\sqrt{5}-1}{2}\thickapprox 0.618$ 黄金比と呼ぶ)であり,著名な「ミロのビーナス」はこの例である.他に,最も美しい人体の頭の頂上からのどまでの長さとのどからへそまでの長さの 比は$\frac{\sqrt{5}-1}{2}$である.もしある人が,上述の2つの黄金比を満たしかつ足の長さが$105 \mathrm{cm}$ 頭の頂上から首の下端までの長さが$26 \mathrm{cm}$のとき身長は概ね
$(A) \ 165 \mathrm{cm}$
$(B) \ 175 \mathrm{cm}$
$(C) \ 185 \mathrm{cm}$
$(D) \ 190 \mathrm{cm}$


$5. \ $関数
$f(x)=\dfrac{\sin x +x}{\cos x +x^2}, \ \ [-\pi,\pi]$
のグラフは概ね



$6. \ $ 我が国の古代図書「周易」では,「卦」で万物の変化を記述する.「複卦」は下から上に並んだ6個の爻(こう)よりなり,爻は 陽爻「」 陰爻「 」よりなる.図は一つの複卦である.

すべての複卦から一つの複卦を選んだ時 複卦の中にちょうど3つの陽爻がある確率は

$(A) \ \dfrac{5}{16} $
$(B) \ \dfrac{11}{32}$
$(C) \ \dfrac{21}{32}$
$(D) \ \dfrac{11}{16}$

$7. \ $ 零ベクトルでない2つのベクトル$\boldsymbol{a},\boldsymbol{b}$が $|\boldsymbol{a}|=2|\boldsymbol{b}|$かつ$(\boldsymbol{a}-\boldsymbol{b})\perp \boldsymbol{b}$ を満たしている.
このとき$\boldsymbol{a},\boldsymbol{b}$のなす角は
$(A) \ \dfrac{\pi}{6}$
$(B) \ \dfrac{\pi}{3}$
$(C) \ \dfrac{2\pi}{3}$
$(D) \ \dfrac{5\pi}{6}$

$8. \ $図は$\dfrac{1}{2+\dfrac{1}{2+\dfrac{1}{2}}}$を求める流れ図である.

図の空欄の中に入るのは
$(A) \ A=\dfrac{1}{2+A}$
$(B) \ A=2+\dfrac{1}{A}$
$(C) \ A=\dfrac{1}{1+2A}$
$(D) \ A=1+\dfrac{1}{2A}$

$9. \ S_n$は等差数列$\{a_n\}$の初項から第$n$項までの和である.
$S_4=0,a_5=5$
のとき
$(A) \ a_n=2n-5$
$(B) \ a_n=3n-10$
$(C) \ S_n=2n^2-8n$
$(D) \ S_n=\dfrac{1}{2}n^2-2n$

$10. \ $楕円$C$の焦点は$F_1(-1,0),F_2(1,0)$,$F_2$を通る直線と$C$との交点を$A,B$とする.
$|AF_2|=2|F_2B|,|AB|=|BF_1|$
のとき,$C$の方程式は
$(A) \ \dfrac{x^2}{2}+y^2=1$
$(B) \ \dfrac{x^2}{3}+\dfrac{y^2}{2}=1$
$(C) \ \dfrac{x^2}{4}+\dfrac{y^2}{3}=1$
$(D) \ \dfrac{x^2}{5}+\dfrac{y^2}{4}=1$

$11. \ f(x)=\sin |x|+|\sin x|$に関する以下の4つの結論:
@ $f(x)$は偶関数
A $f(x)$は区間$(\frac{\pi}{2},\pi)$で単調増加
B $f(x)$は$[-\pi,\pi]$で4個の零点を持つ
C $f(x)$の最大値は$2$
のうちで正しい番号のすべては
$(A) \ $@AC
$(B) \ $AC
$(C) \ $@C
$(D) \ $@B

$12. \ $三角錐$P-ABC$の4頂点は球$O$の球面上にあり,$PA=PB=PC$,$\triangle ABC$は1辺の長さ$2$ の正三角形である.さらに,$E,F$をそれぞれ$PA,AB$の中点とすると$\angle CEF=90^{\circ}$である.
このとき,球$O$の体積は
$(A) \ 8\sqrt{6}\pi$
$(B) \ 4\sqrt{6}\pi$
$(C) \ 2\sqrt{6}\pi$
$(D) \ \sqrt{6}\pi$

空所補充問題で4問よりなる.配点は各問5点合計20点.答えを問題の横線上に記入しなさい.


$13. \ $ 曲線$y=3(x^2+x)e^x$の点$(0,0)$における接線の方程式は$\underline{\hspace{3cm}}$.

$14. \ S_n$は等比数列$\{a_n\}$の初項から第$n$項までの和である.
$a_1=\dfrac{1}{3},a_4^2=a_6$
のとき,$S_5=\underline{\hspace{3cm}}$.

$15.$ 甲,乙2つのチームがバスケットボールの決勝戦を行った. 7番勝負4勝制(どちらかのチームが4勝したとき,そのチームを勝利とし,決勝戦を終了する).
前期の試合の成績より,甲チームのホーム,アウェイの順番は「ホーム,ホーム,アウェイ,アウェイ,ホーム,アウェイ,ホーム」 となった.甲チームのホームでの勝つ確率は$0.6$,アウェイでの勝つ確率は$0.5$で,各会場での試合の結果は 互いに独立である.この時,甲チームが4勝1敗で優勝する確率は$\underline{\hspace{3cm}}.$

$16. \ $双曲線$C:\dfrac{x^2}{a^2}-\dfrac{y^2}{b^2}=1 \ (a > 0,b > 0)$の左,右焦点をそれぞれ$F_1,F_2$とする.$F_1$を通る直線と $C$の2本の漸近線との交点をそれぞれ$A,B$とする.もし
$\overrightarrow{F_1A}=\overrightarrow{AB},\overrightarrow{F_1B}\cdot\overrightarrow{F_2B}=0$
のとき,$C$の離心率は$\underline{\hspace{3cm}}$.

総合問題.配点は合計80点.解答は演算の段取りや証明の過程を文字で説明せよ.

$17.$(配点12点)
$\triangle ABC$の内角$A,B,C$の対辺をそれぞれ$a,b,c$とし,
$(\sin B -\sin C)^2=\sin ^2A-\sin B\sin C$
をみたす.
$(1) \ A$を求めよ.
$(2) \ \sqrt{2}a+b=2c$を満たすとき$\sin C$を求めよ.

$18.$(配点12点)
図のよう直四角柱$ABCD-A_1B_1C_1D_1$があり,底面はひし形である.
$AA_1=4,AB=2,\angle BAD=60^{\circ},$であり,$E,M,N$はそれぞれ$BC,BB_1,A_1D$の中点である.
$(1) \ MN$‖平面$C_1DE$を証明せよ.
$(2) \ $2面角$A-MA_1~-N$の正弦値を求めよ.(2面$AMA_1,NMA_1$のなす角の正弦値)  

$19.$(配点12点)
放物線$C:y^2=3x$の焦点を$F$,傾き$\dfrac{3}{2}$の直線$l$と$C$の交点を$A,B$,$x$軸との交点を$P$とする.
$(1) \ |AF|+|BF|=4$のとき$l$の方程式を求めよ.
$(2) \ \overrightarrow{AP}=3\overrightarrow{PB}$のとき$|AB|$を求めよ.

$20.$(配点12点)
$f(x)=\sin x-\log (1+x)$,$f'(x)$は$f(x)$の導関数である.次を証明せよ.
$(1) \ f'(x)$は区間$(-1,\frac{\pi}{2})$でただ一つの極大値点を取る.
$(2) \ f(x)$はただ2つの零点を取る.

$21.$(配点12点)
ある病気の新薬甲,乙のいずれの薬がより有効であるかを知るために動物試験を行った.試験の方法は次のとおりである: 毎回2匹のハツカネズミを選び,薬の比較試験を行う.2匹のハツカネズミに対して, 無作為に1匹を甲薬に施し,もう1匹に乙薬を施す. 1回の治療結果が出た後,次の試験を行う.これらの試験で,片方の薬によって治癒したハツカネズミがもう一方の薬により治癒したハツカネズミより4匹多いときこの試験を停止し, 治ったハツカネズミが多い方の薬をより有効であると考える.この問題を記述するために次の様に定める:毎回の試験で,もし,甲薬を施したハツカネズミが治癒しかつ乙薬を施したハツカネズミは治癒しなかったとき, 甲薬は$1$点を得, 乙薬は$-1$を得るとする.もし乙薬を施したハツカネズミが治癒しかつ甲薬を施したハツカネズミは治癒しなかったとき,乙薬は$1$点を得, 甲薬は$-1$を得るとする.もし,ともに治癒するか,またはともに治癒しなかったとき,2つの薬はともに$0$点を得るとする.
甲,乙2薬のそれぞれの治癒率はそれぞれ$\alpha,\beta$,1回の試験で甲薬の得る点数を$X$とする.
$(1) \ X$の分布を求めよ.
$(2) \ $甲薬,乙薬ともに試験開始前に4点与える.$p_i \ (i=0,1,\dots ,8)$は 「甲薬が合計で得た点数が$i$点のときの最後に甲薬が乙薬に比べて有効であると判断できる」確率である.このとき,
$p_0=0,p_8=1,p_i=ap_{i-1}+bp_i+cp_{i+1} \ (i=1,2,\dots ,7)$
ただし,$a=P(X=-1),b=P(X=0),c=P(X=1)$
$\alpha =0.5,\beta =0.8$とする.
(@) $\{p_{i+1}-p_i\} \ (i=0,1,2,\dots ,7)$は等比数列であることを証明せよ.
(A) $p_4$の値を求め,$p_4$の値を根拠にこの試験方法の合理性を判断せよ.

$22.$(配点10点)
直角座標系$xOy$において,曲線$C$の媒介変数表示は
$\begin{cases}x=\dfrac{1-t^2}{1+t^2}\\y=\dfrac{4t}{1+t^2}\end{cases} \ \ (t\text{は媒介変数})$
である.座標原点$O$を極点,$x$軸の正の部分を始線とする極座標系で,直線$l$の極座標方程式は
$2\rho \cos \theta +\sqrt{3}\rho \sin \theta +11=0$
である.
$(1) \ C$と$l$の直角座標方程式を求めよ.
$(2) \ C$上の点と$l$との距離の最小値を求めよ.

$23.$(配点10点)
$a,b,c$は正数で$abc=1$を満たす.次を証明せよ.
(1) $\dfrac{1}{a}+\dfrac{1}{b}+\dfrac{1}{c}\leqq a^2+b^2+c^2$
(2) $(a+b)^3+(b+c)^3+(c+a)^3 \geqq 24$

(受験生は必ず答えを解答欄に記入すること.答案用紙上のそれ以外の解答は無効である)



以上が2021年度の問題です.年々難しくなっているような気がします.
優れた問題が多く,日本の高校生にも良い練習になるでしょう.
日本の受験雑誌や入試問題集で扱われる問題は,難関大の比較的難しい問題が多く,もちろんよく練られた優れた問題なのですが, 多くの高校生には難しすぎます.自分の学力を超えた問題の解説解答を読んでも実力はつきません.
自分の学力にあった問題を時間をかけて解き,使った内容の知識の理解を深めていけば, 受験は比較的容易にクリアーできるものです.
さて,解答は公開されているので,ここで述べる必要はないが, 答だけでは不親切と思われる問題や,日中の教え方の違いからか,記述式の解答も 日本の高校生には奇異と思われる場合もあり, 必要と思われる解答を載せておいた.解答例の一つとして扱ってほしい.


<解答例>
$10. \ AF_2=2BF_2,AB=BF_1$
さらに,楕円であるから
$AF_1+AF_2=BF_1+BF_2$
したがって
$AF_1=AF_2$
楕円の方程式を
$\dfrac{x^2}{a^2}+\dfrac{y^2}{b^2}=1$
とおくと
$A(0,b)$
としてよい.
$B$は線分$AF_2$を$3:1$に外分する点であるから
$B'(\dfrac{3}{2},\dfrac{b}{2})$
$B$が楕円上にあるから
$\dfrac{\frac{9}{4}}{a^2}+\dfrac{\frac{b^2}{4}}{b^2}=1$
したがって,
$a^2=3,$
$a^2-b^2=1$
より$b^2=2$


$12.$
まず,$\angle CEF=90^{\circ}$より稜辺の長さを求める.
$PA=a$とおき$\triangle CEF$に注目.
$EF=\dfrac{a}{2},FC=\sqrt{3}$
$CE$は中線定理を利用する.
$CE=x$とおくと
$a^2+2^2=2(x^2+\dfrac{a^2}{4})$
$x^2=\dfrac{a^2}{4}+2$
三平方の定理より
$\dfrac{a^2}{4}+2+\dfrac{a^2}{4}=3$
$\therefore a=\sqrt{2}$
$P$から面$ABC$に下ろした垂線の足を$G$とすると,$G$は$\triangle ABC$の重心.
$AG=\dfrac{2}{\sqrt{3}},PG=\dfrac{\sqrt{2}}{\sqrt{3}}$
$PG < AG$であるから,三角錐$P-ABC$の外接球の中心$O$は線分$PG$の$G$への延長上にある.
$GO=x$とおくと,$OP=OA$より
$\left(\dfrac{\sqrt{2}}{\sqrt{3}}+x \right)^2=\left(\dfrac{3}{\sqrt{2}}\right)^2+x^2$
より
$x=\dfrac{1}{\sqrt{6}}$
$\therefore OP=\dfrac{3}{\sqrt{6}}=\dfrac{\sqrt{3}}{\sqrt{2}}$
外接球の体積を$V$とおくと
$V=\dfrac{4}{3}\cdot \pi \cdot \left(\dfrac{\sqrt{3}}{\sqrt{2}}\right)^3=\sqrt{6}\pi$


$15$
甲チームが4勝1敗で優勝するのは4試合終了後3勝1敗でその後甲が勝つ場合である.
4試合終了後3勝1敗となるのは次の場合である.
$\begin{matrix} \text{ホーム}&\text{ホーム}&\text{アウェイ}&\text{アウェイ}\\ \bigcirc &\bigcirc &\bigcirc &\times \\ \bigcirc &\bigcirc &\times &\bigcirc \\ \bigcirc &\times &\bigcirc &\bigcirc \\ \times &\bigcirc &\bigcirc &\bigcirc \end{matrix} $
$\bigcirc$は勝,$\times$は負
したがって,
$\{\left(\dfrac{6}{10}\right)^2\cdot \left(\dfrac{5}{10}\right)^2\times 2+\dfrac{6}{10}\cdot \dfrac{4}{10}\cdot \left(\dfrac{5}{10}\right)^2\times 2\}\cdot \dfrac{6}{10}$
$=\dfrac{9}{50}$


$16$
$\overrightarrow{F_1B}\cdot \overrightarrow{F_2B}=0$
より
$\angle F_1BF_2=\dfrac{\pi}{2}$
したがって,$B$は線分$F_1F_2$を直径とする円上にある.
$F_2(c,0),\angle BOF_2=\theta$
とおくと
$B(c\cos \theta ,c\sin \theta)$
$A$は線分$F_1B$の中点であるから
$A(\dfrac{c\cos \theta -c}{2},\dfrac{c\sin \theta}{2})$
直線$OA$の傾きは$-\tan \theta$であるから
$\dfrac{c\sin \theta}{c\cos \theta -c}=-\dfrac{\sin \theta}{\cos \theta}$
したがって
$\cos \theta =\dfrac{1}{2}$
$\dfrac{b}{a}=\tan \theta =\sqrt{3}$
$b=\sqrt{3}a,c=2a$
したがって,離心率$e$は
$e=\dfrac{c}{a}=2$


$18.$
$(1) \ \ AD$の中点を$F$とすると
$\overrightarrow{MN}$‖$\overrightarrow{FB}$(‖$\overrightarrow{DE}$)‖面$C_1DE$
より成り立つ.
$(2) \ $定義にしたがって,2面のなす角を求める.
まず,底面が正三角形$ABD$の三角柱で考えればよいことに注意する.
$\angle AMA_1=90^{\circ}$
であるから,$\triangle A_1MD$で辺$A_1D$上に点$L$を
$\angle A_1ML =90^{\circ}$
となるように点$L$を取り,$\angle AML$が求める2平面のなす角である.
$\angle MA_1D=\alpha$とおくと
$\cos \alpha =\dfrac{\sqrt{5}}{2\sqrt{2}},\sin \alpha =\dfrac{\sqrt{3}}{2\sqrt{2}}$
(∵ $\triangle MA_1D$は二等辺三角形)
より
$A_1L=\dfrac{8}{\sqrt{5}},DL=\dfrac{2}{\sqrt{5}},ML=\dfrac{2\sqrt{6}}{\sqrt{5}}$
次に$AL$であるが,$AA_1:AD=2:1$で$A_1L:LD=4:1$であるから,$L$は$A$から$A_1D$ に下ろした垂線の足.したがって
$AL=2DL=\dfrac{4}{\sqrt{5}}$
$AM=2\sqrt{2}$
であるから
$\cos \theta =\dfrac{\dfrac{24}{5}+8-\dfrac{16}{5}}{2\cdot \dfrac{2\sqrt{6}}{\sqrt{5}}\cdot 2\sqrt{2}}=\dfrac{\sqrt{3}}{\sqrt{5}}$
$\therefore \sin \theta =\dfrac{\sqrt{2}}{\sqrt{5}}$

もちろん2平面の法線ベクトルを用いて解いてもよい.この方が単純な計算で面倒なところがない.
別解 $A$を原点,$AB$を$x$軸,$AA_1$を$z$軸とする直交座標系で考える.
$A(0,0,0),B(2,0,0),D(2\cos 60^{\circ},2\sin 60^{\circ},0)$
より$D(1,\sqrt{3},0),$および
$A_1(0,0,4),B(2,0,4),D_1(1,\sqrt{3},4)$
面$AMA_1,DMA_1$の法線ベクトルをそれぞれ$\overrightarrow{n_1},\overrightarrow{n_2}$とすると,
$\overrightarrow{n_1}=(0,1,0)$($zx$平面)
次に
$\overrightarrow{DA_1}=(-1,-\sqrt{3},4),\overrightarrow{DM}=(1,-\sqrt{3},2)$
$\overrightarrow{n_2}=\overrightarrow{DA_1}\times \overrightarrow{DM} =(2\sqrt{3},6,2\sqrt{3})$‖$(1,\sqrt{3},1)$
したがって
$\cos \theta =\dfrac{\sqrt{3}}{1 \cdot \sqrt{5}}$
$\therefore \sin \theta =\dfrac{\sqrt{2}}{\sqrt{5}}$


参考
空間図形で,2つのベクトルに垂直なベクトル(平面の法線ベクトル)を求めるとき外積が便利であるが, 直接計算しても難しくはない.
$\overrightarrow{DA_1}=(-1,-\sqrt{3},4),\overrightarrow{DM}=(1,-\sqrt{3},2)$
に垂直なベクトルを$\overrightarrow{n_2}=(x,y,z)$とおくと
$-x-\sqrt{3}y+4z=0 \ \ \cdots (*_1)$
$x-\sqrt{3}y+2z=0 \ \ \cdots (*_2)$
$(*_1)+(*_2)$より$y=\sqrt{3}z$
$(*_1)-(*_2)$より$x=z$
したがって$x:y:z=1,\sqrt{3}:1$
$\overrightarrow{n_2}=(1,\sqrt{3},1)$を取ればよい.


$19.(1) $線分$AB$の中点を求めるのが分かりやすい.
$A,B$の$y$座標をそれぞれ$t,s \ (t > 0 ,s < 0)$すなわち
$A\left(\dfrac{t^2}{3},t\right),B\left(\dfrac{s^2}{3},s\right)$
と置く.
$AF=\dfrac{t^2}{3}+\dfrac{3}{4},BF=\dfrac{s^2}{3}+\dfrac{3}{4}$
であるから,$AF+BF=4$より
$\dfrac{1}{2}(\dfrac{t^2}{3}+\dfrac{s^2}{3})=\dfrac{5}{4}$
直線$AB$の傾きが$\frac{3}{2}$および
$\dfrac{t-s}{\frac{t^2}{3}-\frac{s^2}{3}}=\dfrac{3}{t+s}$
より
$t+s=2$
したがって,$AB$の中点の座標は$(\frac{5}{4},1)$
$l$の方程式は
$y-1=\dfrac{3}{2}(x-\dfrac{5}{4})$
$(2)$
$A(3t^2,3t),B(\dfrac{t^2}{3},-t)$
と置くことができる.
直線$AB$の傾きは
$\dfrac{3t-(-t)}{3t^2-\frac{t^2}{3}}=\dfrac{3}{2t}$
より$t=1$
$\therefore A(3,3),B(\frac{1}{3},-1)$
$AB^2=\left(\dfrac{8}{3}\right)^2+4^2$
$=4^2\left(\dfrac{4}{9}+1\right)=\dfrac{4^2\cdot 13}{3^2}$
$\therefore AB=\dfrac{4\sqrt{13}}{3}$


$20.$これは2次導関数まで利用するなかなかの難問です.
$f(x)=\sin x -\log (1+x)$
$f'(x)=\cos x -\dfrac{1}{1+x}$
$f''(x)=-\sin x +\dfrac{1}{(1+x)^2}$
$f'''(x)=-\cos x -\dfrac{2}{(1+x)^3}$
$(1) \ f'(x)$の$(-1,\frac{\pi}{2}]$における増減表は次の様になる.
$\begin{array}{|c|c|c|c|c|}\hline x & -1 & \cdots & x_0 & \cdots & \frac{\pi}{2} \\ \hline f''(x) & +\infty & + & 0 & - & - \\ \hline f'''(x) & & - & - & - & - \\ \hline f'(x) & -\infty & \nearrow &f'(x_0) & \searrow & \\ \hline \end{array}$

($-\frac{\pi}{2} < -1 < x < \frac{\pi}{2}$より$f'''(x) < 0$.したがって$f''(x)$は単調減少.
さらに$\displaystyle \lim_{x\to -1+0}f''(x)=+\infty$,$f''(\frac{\pi}{2}) < 0$ であるから, 区間$(-1,\frac{\pi}{2})$に$f''(x_0)=0$を満たす$x_0$がただ一つ存在する.それで上の増減表が出来上がる)
$f'(x)$は増減表よりただ一つの極大値点を持つ.
$(2)$
$(1)$で求めた$f'(x)$の増減より$f'(x)$の符号を調べる.
$f''(0) > 0$より$0 < x_0$である.
さらに$f'(0)=0$より$f'(x_0) > 0$
$x_0 < x < \frac{\pi}{2}$で$f'(x)$は単調減少で((1)の増減表),$f'(\frac{\pi}{2}) < 0$であるから
$f'(x_1)=0 \ (x_0 < x_1 < \frac{\pi}{2})$
を満たす$x_1$が存在する.
さらに,$\frac{\pi}{2} < x < \pi$のとき
$f'(x) < 0$であるから$f(x)$の区間$(-1,\pi]$における増減表は次の様になる.
$\begin{array}{|c|c|c|c|c|c|c|c|c|c|c|c|}\hline x & -1 & \cdots & 0 & \cdots & x_0 & \cdots & x_1 & \cdots &\frac{\pi}{2} & \cdots & \pi\\ \hline f'(x) & & - & 0 & + & + & + &0 &-&-&-&-\\ \hline f(x) & & \searrow &0 & \nearrow &f(x_0) & \nearrow & f(x_1) &\searrow &f(\frac{\pi}{2}) &\searrow &f(\pi) \\ \hline \end{array}$

$f(\frac{\pi}{2})=1-\log(1+\frac{\pi}{2})$

$1+\dfrac{\pi}{2} < 1+1.6 < e$
より
$f(\frac{\pi}{2}) > 0$
であるから区間$(-1,\pi]$の区間の増減表を考える必要がある.
$f(\pi)=-\log (1+\pi) < 0 $
増減表より$(-1,\pi)$でただ2つの零点をとる.
最後に,区間$(\pi , +\infty)$を調べる.
$\log (1+x) \geqq \log (1+\pi) \geqq \log (1+3) > 1$
より$f(x) < 0$であるから$f(x)$は零点を持たない.
以上で$f(x)$はただ2つの零点を取ることが証明された.

$21.$
$(1) \ X$の確率分布は次の通り.
  $\begin{array}{|c|c|c|c|}\hline X & -1 & 0 & +1 \\ \hline P & (1-\alpha)\beta & \alpha \beta +(1-\alpha )(1-\beta) & \alpha (1-\beta) \\ \hline \end{array}$

$(2)$問題の意味を読み取ることが大切です.
何回か試験を行ったとき,甲が$i$点の時,その後試験を繰り返し甲薬の方が乙薬より有効であると判断できる確率を$p_i$と置いたわけです.
$p_0$は乙薬が有効と判断できるので$p_0=0$で,$p_8$は甲薬が有効と判断できるので$p_8=1$です.
$p_1,\dots p_7$はまだわかりませんが,関係式($p_i$の漸化式)を作り$p_i$を求めます.
問題の中に関係式があり,この関係式がこの問題の本質です.
関係式は次の様に考えます.
甲の得点が$i$のとき,次の試験によって
甲が有効と出たら甲の得点は$i+1$その時の甲が有効と判断できる確率は$p_{i+1}$
甲と乙の有効性が同じと出たら甲の得点は$i$その時の甲が有効と判断できる確率は$p_{i}$
乙が有効と出たら甲の得点は$i-1$その時の甲が有効と判断できる確率は$p_{i-1}$
したがって
$p_i=P(X=-1)p_{i-1}+P(X=0)p_i+P(X=1)p_{i+1}$
これが問題文の中の関係式の意味です.
$\alpha =0.5,\beta =0.8$のとき,確率変数$X$の確率分布は
$\begin{array}{|c|c|c|c|}\hline X & -1 & 0 & +1 \\ \hline P & \dfrac{4}{10} &\dfrac{5}{10} & \dfrac{1}{10} \\ \hline \end{array}$
であるから$\{p_i\}$の漸化式は
$p_i=\dfrac{4}{10}p_{i-1}+\dfrac{5}{10}p_i+\dfrac{1}{10}p_{i+1}$
整理すると
$p_{i+1}-5p_i+4p_{i-1}=0,p_0=0,p_8=1$
2次方程式
$x^2-5x+4=0$
を解いて
$x=1,4$
したがって,与えられた漸化式は
$a_{i+1}-a_i=4(a_i-a_{i-1}) \ \cdots (\sharp)$
$a_{i+1}-4a_i=(a_i-4a_{i-1}) \ \cdots (\flat)$
$a_1=c$とおくと
$(\sharp )$より
$a_{i+1}-a_i=4^i(a_1-a_0)=4^ic$
$(\flat )$より
$a_{i+1}-4a_i=a_1-4a_0=c$
上の式から下の式を引き
$3a_i=c(4^i-1)$
$a_8=1$より$c=\dfrac{3}{4^8-1}$
$\therefore a_i=\dfrac{4^i-1}{4^8-1}$
$p_4=\dfrac{4^4-1}{4^8-1}=\dfrac{1}{4^4+1}=\dfrac{1}{257}$
$p_4$は甲,乙4点のときすなわち試験の開始前の状態である.
$\alpha =\frac{5}{10},\beta =\frac{8}{10}$であるから乙薬の方が甲薬より有効であると判断できる試験が合理的である.
この試験で,反対の結果すなわち,甲が有効である結果を得る確率は概ね$\frac{4}{1000}$. これは無視できる値である.したがって,この試験方法は合理的な方法であると考えられる.



$22$いろいろな解答がある.
$\cos 2\theta=\dfrac{\cos ^2\theta -\sin ^2\theta}{\cos ^2\theta+\sin ^2 \theta} =\dfrac{1-\tan ^2 \theta}{1+\tan ^2 \theta}$
$\sin 2\theta=\dfrac{2\sin \theta \cos \theta}{\cos ^2\theta+\sin ^2 \theta} =\dfrac{2\tan \theta}{1+\tan ^2 \theta}$
より
$t=\tan \dfrac{\theta}{2}$
とおくと
$x=\cos \theta,y=2\sin \theta$
したがって,$C$の直角座標方程式は
$x^2+\dfrac{y^2}{4}=1$
次の様に解くことができる.
媒介変数表示が
$x=f(t,t^2),y=g(t,t^2)$
のとき,$t,t^2$をそれぞれ$x,y$で表し
$(t)^2=t^2$
に代入する.
この問題では
$x=\dfrac{1-t^2}{1+t^2}$
より
$t^2=\dfrac{1-x}{1+x}$
さらに
$y=\dfrac{4t}{1+t^2}$
より
$t=\dfrac{y}{2(1+x)}$
したがって,
$\left(\dfrac{y}{2(1+x)}\right)^2=\dfrac{1-x}{1+x}$
を計算し
$x^2+\dfrac{y^2}{4}=1$

極方程式
$2\rho \cos \theta +\sqrt{3}\rho \sin \theta +11=0$
の直角座標方程式は
$x=\rho \cos \theta,y=\rho \sin \theta$
より
$2x+\sqrt{3}y+11=0$
$(2)$は$P(\cos \theta,2\sin \theta)$と直線$2x+\sqrt{3}y+11=0$との距離$d$
$d=\dfrac{|2\cos \theta +2\sqrt{3}\sin \theta+11|}{\sqrt{7}}$
の最小値を求めればよい.
$2^2+(2\sqrt{3})^2=16$
より最小値は
$\dfrac{-4+11}{\sqrt{7}}=\sqrt{7}
$
また,傾きが$-\frac{2}{\sqrt{3}}$となる楕円の接線の接点を求めて,最小値を求めてもよい.
その方法で解くと
楕円
$x^2+\dfrac{y^2}{4}=1$
の$P(x_0,y_0)$における接線は
$x_0x+\dfrac{y_0y}{4}=1$
傾きが$-\frac{2}{\sqrt{3}}$より
$\begin{cases}-\dfrac{4x_0}{y_0}=-\dfrac{2}{\sqrt{3}}\\ x_0^2+\dfrac{y_0^2}{4}=1\end{cases}$
を解けばよい.
これを解けば
$(x_0,y_0)=(\dfrac{1}{2},\sqrt{3}),(-\dfrac{1}{2},-\sqrt{3})$
グラフからわかるように,$(-\dfrac{1}{2},-\sqrt{3})$が最小値を与える点で,もう一方は最大値 を与える点である.
点$(-\dfrac{1}{2},-\sqrt{3})$と直線$2x+\sqrt{3}y+11=0$との距離を求め $\sqrt{7}$

$23$
$(1) \ \ a^2+b^2+c^2-\dfrac{1}{a}-\dfrac{1}{b}-\dfrac{1}{c}$
$=a^2+b^2+c^2-bc-ca-ab$
$=\dfrac{1}{2}((a-b)^2+(b-c)^2+(c-a)^2)\geqq 0$
等号が成り立つのは$a=b=c=1$
$(3)$
$a^3+b^3+c^3-3abc=(a+b+c)(a^2+b^2+c^2-ab-bc-ca)$
より$a > 0,b > 0 c > 0$のとき
$a^3+b^3+c^3\geqq 3abc$
問題で$a > 0,b > 0 c > 0$であるから
$(a+b)^3+(b+c)^3+(c+a)^3\geqq 3(a+b)(b+c)(c+a)$
(この不等式は3変数の相加平均と相乗平均の関係とみてもよい)
相加平均,相乗平均の関係より
$\geqq 3\cdot 2\sqrt{ab}\cdot 2\sqrt{bc}\cdot 2\sqrt{ca}$ $=24abc=24$
等号が成り立つのは第1の不等式でも,第2不等式でも$a=b=c=1$であるから,等号が成り立つのは
$a=b=c=1$


(2021年8月記す)


目次